LSAT and Law School Admissions Forum

Get expert LSAT preparation and law school admissions advice from PowerScore Test Preparation.

 Administrator
PowerScore Staff
  • PowerScore Staff
  • Posts: 8917
  • Joined: Feb 02, 2011
|
#59636
Complete Question Explanation
(The complete setup for this game can be found here: lsat/viewtopic.php?t=1624)

The correct answer choice is (D)

This is one of the easiest LSAT games questions ever (proving that even if you miss the setup completely, you can still answer some questions correctly in a game just by applying the rules). From the first rule we know that F must serve building Y. Since answer choices (A), (B), and (E) fail to include Y, they are incorrect. From the second rule we know that F must serve exactly two buildings. Since answer choice (C) contains three buildings, it is incorrect.

Answer choice (D): This is the correct answer choice.
 DlarehAtsok
  • Posts: 50
  • Joined: Nov 18, 2015
|
#41777
I am having trouble with the wording of this stem. From the setup, I knew that Falafel must serve Y and one from X and Z, given that the other would have to be served by the pita truck. However I misunderstood the question as it was asking about the possible office buildings that the truck could serve at any time, not a particular combination, so I chose C. How would the question stem be, if this would be a List question (so I may understand the difference)?

Thank you!
 James Finch
PowerScore Staff
  • PowerScore Staff
  • Posts: 943
  • Joined: Sep 06, 2017
|
#41960
Hi Dlareh,

This is a very common question stem for Grouping games. What the phrasing "could be a complete and accurate list of the [groups a variable is in]" means is that the correct answer choice will have been "solved" for that variable, so when it has an either/or possibility, it must be either in or out for the purposes of the question.

Here, as you note, we know that variable F must go into group Y, in every circumstance, which leaves only answer choices (C) and (D). Then a second rule, combined with the rule that all variables must appear in at least one group, says that variable P cannot appear in any group in which F appears. Again, you realized this, and recognized that F could appear in either group X or group Z, along with Y, but not both. This means that for a question like this, that asks to solve for the maximum number of groups possible for a particular variable, the correct answer choice will have placed the variable in one of the two possible groups. Here, that is group Z, so (D) ends up as the correct answer choice.

Hope this clears things up!
 haganskl
  • Posts: 43
  • Joined: May 30, 2019
|
#76819
Hello. I got this question wrong too.
What if the question was “What is a list of buildings the F truck could visit? Would this question allow for answer choice C?

TIA
 Adam Tyson
PowerScore Staff
  • PowerScore Staff
  • Posts: 5153
  • Joined: Apr 14, 2011
|
#76877
That would do it, haganskl! F must visit Y and must serve either X or Z, but not both. So a complete list of all the buildings F could serve would be a list of all three of the trucks.

The difference here is in the use of absolute language vs. the language of possibility. If they wanted a list of all the buildings eligible for all possible solutions to the game, they would ask "which of the following IS a complete and accurate list." If they want just a single possible solution, they would ask "which of the following COULD BE a complete and accurate list."
User avatar
 Cantthinkofagood1
  • Posts: 6
  • Joined: Sep 04, 2023
|
#104320
I knew that the falafel truck had to serve only 2 buildings because of rule 2, but I still got it wrong because I read the question too fast and misunderstood it as the following. "What is a complete and accurate list of the office buildings that the falafel trucks could serve?" Had the question been what I thought it was, then would the answer be C? If so, then the emphasis isn't about "complete and accurate" or "could", but rather about the order which these phrases occur? This is so confusing.
User avatar
 srusty
PowerScore Staff
  • PowerScore Staff
  • Posts: 32
  • Joined: Nov 30, 2023
|
#104353
Cantthinkofagood1 wrote: Wed Dec 06, 2023 2:31 pm I knew that the falafel truck had to serve only 2 buildings because of rule 2, but I still got it wrong because I read the question too fast and misunderstood it as the following. "What is a complete and accurate list of the office buildings that the falafel trucks could serve?" Had the question been what I thought it was, then would the answer be C? If so, then the emphasis isn't about "complete and accurate" or "could", but rather about the order which these phrases occur? This is so confusing.
Hi C,

I understand where you're coming from - this question stem is a bit tricky with some nuance. Technically, our deductions with what we are given tell us that F can serve all X,Y, and Z, but this is not an accurate list as we know it is limited to serving only two trucks. Moving the "could" around to "Which of the following is a complete and accurate list of each of the office buildings that the falafel truck could serve" may lend itself to including X,Y, and Z.

As you correctly stated, the way the question is phrased is asking us to pick the answer choice that is realistic. We know our answer choice has to include Y, and we know it is limited to two trucks, thus answer choice (D) is correct.

Hope this helps!

Get the most out of your LSAT Prep Plus subscription.

Analyze and track your performance with our Testing and Analytics Package.